What Mistake Am I Making in This Limit Calculation?

  • Thread starter Thread starter kLPantera
  • Start date Start date
Click For Summary
The discussion revolves around the limit calculation as x approaches positive infinity for the expression x + r(x^2 + 2x). The original poster attempts to simplify the limit by multiplying by the conjugate but encounters an undefined result, leading to confusion about their calculations. A key point raised is the ambiguity surrounding the variable r, which could be a positive or negative number, affecting the limit's outcome. Additionally, the importance of proper notation and the use of brackets is emphasized to clarify the intended mathematical expressions. The poster is advised to reassess their approach without the conjugate and focus on the behavior of the limit based on the value of r.
kLPantera
Messages
43
Reaction score
0

Homework Statement



limi as x-> positive infinity x+r((x^2)+2x)


The Attempt at a Solution



multiply by conjugate x-r((x^2)+2x)

I get (x^2)-(x^2)+2x/x-r((x^2)+2x)

Which becomes 2x/x-r((x^2)+2x)

Which I end up with 2/1-r(1-(2/x))


But I go wrong somewhere because I end up with 2/0 and the answer is positive infinity. I keep getting undefined and I can't see where I'm going wrong. Could someone point it out?
 
Physics news on Phys.org
kLPantera said:

Homework Statement



limi as x-> positive infinity x+r((x^2)+2x)


The Attempt at a Solution



multiply by conjugate x-r((x^2)+2x)

I get (x^2)-(x^2)+2x/x-r((x^2)+2x)

Which becomes 2x/x-r((x^2)+2x)

Which I end up with 2/1-r(1-(2/x))


But I go wrong somewhere because I end up with 2/0 and the answer is positive infinity. I keep getting undefined and I can't see where I'm going wrong. Could someone point it out?

What is r? Is it a number, a function, or what? If it is a number > 0 you should have no trouble saying what is ## lim_{x \rightarrow \infty} x + r(x^2 + x).## If r is a number < 0 it is almost as easy, but you need to be careful. And, of course, if r = 0 it is easier still.

I have no idea why you would want to take conjugates, and anyway, I stopped reading your work because you have not used brackets I cannot tell whether you mean
x^2 - x^2 + \frac{2x}{x} - r(x^2 + x), (which IS the meaning of what you wrote) or whether you mean
x^2 - x^2 - \frac{2x}{x - r(x^2 + 2x)}
or
x^2 - \frac{x^2 + 2x}{x - r(x^2 + 2x)},
or several other possibilities.

RGV
 
Question: A clock's minute hand has length 4 and its hour hand has length 3. What is the distance between the tips at the moment when it is increasing most rapidly?(Putnam Exam Question) Answer: Making assumption that both the hands moves at constant angular velocities, the answer is ## \sqrt{7} .## But don't you think this assumption is somewhat doubtful and wrong?

Similar threads

  • · Replies 11 ·
Replies
11
Views
2K
  • · Replies 2 ·
Replies
2
Views
2K
Replies
5
Views
2K
  • · Replies 9 ·
Replies
9
Views
2K
  • · Replies 25 ·
Replies
25
Views
2K
Replies
4
Views
2K
  • · Replies 6 ·
Replies
6
Views
2K
  • · Replies 6 ·
Replies
6
Views
2K
  • · Replies 10 ·
Replies
10
Views
2K
  • · Replies 8 ·
Replies
8
Views
2K